Which of the following
is most likely the next step in the series?

Which Of The Followingis Most Likely The Next Step In The Series?

Answers

Answer 1

Answer:

B.

Step-by-step explanation:

The numbers are always in the blue side, and the blue side starts at facing right, left, right, and then left.

Answer 2
b. is the answer for the questions

Related Questions

If the quadratic formula is used to find the solution set of 3x + 4x-2 = 0, what are the solutions? ​

Answers

Answer is c
hope it helps I’m new at this

Find the area of the triangle whose vertices are (2, 3); (-1, 0); (2, -4)...
[Also show the steps of the solution]
Please answer correctly, it's really urgent!​

Answers

Answer:

Area = 14.5

Step-by-step explanation:

Let's label the given coordinates A, B, C;

Thus;

A = (2, 3)

B = (-1, 0)

C = (2, -4)

From the coordinate geometry formula, the formula for area of a triangle with 3 vertices is;

Area = [Ax(By - Cy) + Bx(Cy - Ay) + Cx(Ay - By)]/2

Area = [2(0 - (-4)) + (-1)(-4 - 3) + 2(3 - (-4))]/2

Area = 29/2

Area = 14.5

© Find the quotient of 3/8 and ,4/9
Give your answer as a fraction in its simplest form.

Answers

[tex] \frac{3}{8} \div \frac{4}{9} \\ = \frac{3}{8} \times \frac{9}{4} \\ = \frac{27}{32} [/tex]

Hope it helps!!!

Thanks!!!

Hey Guys! Can You Please Help Me In My Math​

Answers

This would be a great tree diagram
Sorry...

pic is not clear.....

The a oranial price of a skate broad was reduced by 15 dollars .the new price is 49 dollars if p=the stakebroads oranail price in dollars what mathematical sentence expresses the information
a.49-p=15
b.15-p=49
c.15+p=49
d.p-15=49

Answers

Answer:

d. p - 15 = 49

Step-by-step explanation:

p = original price

The price was reduced by 15 dollars.

The new price is p - 15.

The new price is 49.

p - 15 = 49

Answer: d. p - 15 = 49

Can someone help me?

Answers

Answer:

C

Step-by-step explanation:

its asking for y, on the graph, the line is placed on point 4

Solve simultaneously

4X + 3y= 19
2X +5y= 20

Answers

answer

4

2

Step-by-step explanation:

4x+3y=19

4x=19-3y

4x=16

x=16/4

x=4

2x+5y=20

5y=20/2x

5y= 10

y=10/5

y=2

Find the measure of the missing angle using exterior angle sum theorm

Answers

35 degrees.

Assuming 145 is the exterior angle, the interior is 35. 180-145=35
Add 110 to 35 and get 145 degrees. Which leads the remaining angle 35 degrees

Answer:

35

Step-by-step explanation:

The exterior sum theorem states the exterior angle is equal to the sum of the opposite interior angles

145 = ?+110

145 - 100 = ?

35 = ?

please tel me answer of under root 3+4i
without calculatot with steps

Answers

Hello,

[tex]Let's\ say \\\\z=\sqrt{3+4*i} =a+b*i\\\\z^2=3+4*i=(a+b*i)^2=a^2-b^2+2i*a*b\\\\\\if \ a\neq 0\\\left\{\begin{array}{ccc}a^2+b^2&=&3\\2ab=4\\\end{array}\right.\\\\\\\left\{\begin{array}{ccc}b=\dfrac{2}{a}\\a^2-(\dfrac{2}{a})^2=2\\\end{array}\right.\\\\\\a^4-4=3*a^2\\a^4-3a^2-4=0\\\\\Delta=(-3)^2-4*1*(-4)=25=5^2\\\\a^2=4\ or \ a^2=-1 (impossible)\\\\So:\\(a=2\ and\ b=1)\ or\ (a=-2\ and\ b=-1)\\[/tex]

Roots are thus 2+i and -2-i

There is an other using a geometrical formula (formule de Moivre)

Determine if the ordered pair (-1,-5) is a solution to the inequality y<_ -3/4x-1.

A.) No,because (-1,-5) is above the line
B.) Yes, because (-1,-5) is below the line
C.) No, because (-1,-5) is on the line
D.) Yes, because (-1,-5) is on the line

Answers

Answer:

the answer is B, comment if you need explanation

Step-by-step explanation:

The answer is B come in if you need an explanation

I forgot to label the triangle below! I just know that the cos A = 0.48. Based on this information, which angle should be marked A?

Answers

Answer:

angle 1

Step-by-step explanation:

Using the trigonometric mnemonic SOH CAH TOA, we know that cos or cosine is the ratio between the adjacent side and hypotenuse side.

This means that if cos A = 0.48, A is the measure of the angle which it's relative adjacent side divided by the hypotenuse of the triangle will be around 0.48.

Let's try angle 2, cos (angle 2) = adjacent / hypotenuse = 7.8 / 8.9 = 0.876404494382 ≈ 0.87 ≠ 0.48. Since the proportions are not equal, this angle cannot be the one marked as A.

Since angle 3 is a right angle, the adjacent could be either side so it cannot be correct. Thus angle 1 is correct.

Which best describes the vertex of the graph?


a (-3, -4)
b (-3, -4)
c (3, -4)
d (3, -4)

Answers

Answer: C

Step-by-step explanation:

C. Is the answer for sure

In ΔCDE, the measure of ∠E=90°, ED = 28, CE = 45, and DC = 53. What ratio represents the tangent of ∠C?

Answers

Answer:

3/5

Step-by-step explanation:

prependicular / hypotenuse

sin C=ED/CD

=3/5

You invest $4.000 in a savings account. The account pays 3% annual interest. How much money will be in the savings account after 9 years?

A) $4,938.29
B) $5,219.20
C) $5,124.33
D) $6,003.45​

Answers

Answer:

4000*0.03*9=1080

4000+1080=5080

umm but like thats not one of the answers sorry

Hope This Helps!!!

Step-by-step explanation:

4000*3% = 120

120*9= 1080

1080+4000=5080

a. 1.3
b. 9.6
c. 13
d. 1

Answers

Answer:      I believe the answer would be 1.3

           HOWEVER, if you do have to round to the nearest tenth then the answer would be 1

Step-by-step explanation:    

2x+7 = 12x - 6

(subtract 2x from both sides)

7 = 10x - 6

(add 6 to both sides)

13 = 10x

(divide 13/10)

1.3 = x

The angle of elevation of a tree at a distance of 10m from the foot of the tree is 43°. Find the height of the tree

Answers

Answer:

9.32m is the height of. the tree from the ground.

Question 18 (5 points)
Determine the sum of the measures of the interior angles of a dodecagon (12-sided
polygon).
540°
1,800°
360°
2,160°

Answers

Answer:

its 540 bro

Step-by-step explanation:

All current-carrying wires produce electromagnetic (EM) radiation, including the electrical wiring running into, through, and out of our homes. High frequency EM is thought to be a cause of cancer; the lower frequencies associated with household current are generally assumed to be harmless. The following table summarizes the probability distribution for cancer sufferers and their wiring configuration in the Denver area.

Leukemia Lymphoma Other Cancers
High Frequency wiring 0.242 0.047 0.079
Low frequency wiring 0.391 0.098 ???
(a) What is the missing probability (labelled ???) in the above table?

(b) What is the probability of having high frequency wiring among cancer suffers in the Denver area?

(c) Is the event "Having Leukemia" independent of the event "Having high frequency frequency wiring"? Explain.

Answers

Answer:

[tex]x = 0.143[/tex]

[tex]P(High\ |\ Cancer) = 0.215[/tex]

Not independent

Step-by-step explanation:

Given

See attachment for proper table

Solving (a): The missing probability

First, we add up the given probabilities

[tex]Sum = 0.242+0.047+0.079+0.391+0.098[/tex]

[tex]Sum = 0.857[/tex]

The total probability must add up to 1.

If the missing probability is x, then:

[tex]x + 0.857 = 1[/tex]

Collect like terms

[tex]x = -0.857 + 1[/tex]

[tex]x = 0.143[/tex]

Solving (b): P(High | Cancer)

This is calculated as:

[tex]P(High\ |\ Cancer) = \frac{n(High\ n\ Cancer)}{n(Cancer)}[/tex]

So, we have:

[tex]P(High\ |\ Cancer) = \frac{0.079}{0.242+0.047+0.079}[/tex]

[tex]P(High\ |\ Cancer) = \frac{0.079}{0.368}[/tex]

[tex]P(High\ |\ Cancer) = 0.215[/tex]

Solving (c): P(Leukemia) independent of P(High Wiring)

From the attached table

[tex]P(Leukemia\ n\ High\ Wiring) = 0.242[/tex]

[tex]P(Leukemia) = 0.242 + 0.391 =0.633[/tex]

[tex]P(High\ Wiring) = 0.242+0.047+0.079=0.368[/tex]

If both events are independent, then:

[tex]P(Leukemia\ n\ High\ Wiring) = P(Leukemia) * P(High\ Wiring)[/tex]

[tex]0.242 = 0.633 * 0.368[/tex]

[tex]0.242 \ne 0.232[/tex]

Since the above is an inequality, then the events are not independent

can someone answer plssss gives 100 pints i think bc I picked 100

Answers

[tex]1. \frac{20}{100} [/tex]

[tex]2. \frac{1}{5} [/tex]

Answer:  1

Explanation:

There are a few ways to do this. One way is to notice that the jump from 5 to 100 is "times 20" (go from right to left across the bottom denominators).

So we must do the same "times 20" type of jump when going across the numerators. If x is the numerator for the right hand side, then we go from x to 20. That must mean x = 1

Put another way, we could have these steps

20/100 = x/5

20*5 = 100*x ... cross multiplication

100 = 100x

100x = 100

x = 100/100 .... dividing both sides by 100

x = 1

We see that the fraction 20/100 reduces fully to 1/5

To go from 1/5 to 20/100, we multiply both parts by 20 (divide both parts by 20 to go in reverse).

If and are the zeroes of the polynomial 2x^2+3x+5 then the value of 1/alpha + 1/beta
Pls answer fast..I’ll mark the brainlest

Answers

Answer:

-4/15

Step-by-step explanation:

Mathematically, we know that;

alpha= -b/a and beta = c/a

a refers to the coefficient of the x^2 which is 2

b is the coefficient of x which is 3

c is the last number which is 5

alpha = -b/a = -3/2

beta = c/a = 5/2

1/alpha = -2/3

1/beta = 2/5

so we have the sum as;

-2/3 + 2/5

= -5(2) + 3(2)/15 = (-10 + 6)/15 = -4/15

pls help me in these questions ​

Answers

Answer:

1=85

2=10

3=108

Step-by-step explanation:

Number 1: Calculate each angle...and you know a straight line is 180°. N the interior sum of a quadrilateral is always 360°.

Number2: Use corresponding, alternate and interior angles method. It will help

Number 3: It's just about solving the interior sum of the pentagon

Answer:

2. 10 degrees

3. 108 degrees

Step-by-step explanation:

sorry I reposted it because the previous answer was deleted and plz mark me as a brainliest.

if sam has 20 watermelons and he eats 14 how many does he have left?

Answers

Answer:

6 duh!

you can do this by yourself lol

Instructions: Problem 2 ! Find the missing angle in the image below. Do not include spaces in your answers

Answers

Step-by-step explanation:

since angles in a triangle add up to 180

<vuw=180-(71+23)

=86°

since angles in a straight line add up to 180

<vuf=180-86

=94

Solve for x

Marking brainliest

Answers

Answer:

110°

Step-by-step explanation:

write the equation of the line that passes through the points (1,6) and (9,-8). put your answer in fully reduced slope form, unless it is a vertical or horizontal line.

Answers

Answer:

Y = - 7/4 X + 31/4

Step-by-step explanation:

M = 14/-8 = - 7/4

6 = - 7/4 + B

24 = -7 + 4B

31 = 4B

B = 31/4

If f(x) = 3x + 10 and g(x) = 2x - 4, find (f+g)(x).
O A. (f+g)(x) = -34 - 2x - 14
B. (f+ g)(x) = 3x - 2x + 14
O C. (f+ g)(x) = 5x + 6
D. (f+ g)(x) = 3x + 2x + 6

Help! I’m getting really confused which one it is

Answers

Answer:

O D

Step-by-step explanation:

f+g(x)

3x+10+2x-4

3x +2x+6

Plis help me it’s for today

Answers

Answer:

Following are the solution to the given points:

Step-by-step explanation:

For question 1:

[tex]\to 3^{-4}= \frac{1}{3^4}=\frac{1}{81}=0.0123456789[/tex]

For question 2:

[tex]\to (-2)^{3}\cdot(-2)^{4}\cdot(-2)^{-1}=-8\cdot-16\cdot -\frac{1}{2}= 128\cdot -\frac{1}{2}=-64[/tex]

For question 3:

[tex]\to 7^{-4} \div 7^{-2}= \frac{1}{7^{4}} \div \frac{1}{7^{2}}=\frac{1}{7^{4}} \times \frac{7^{2}}{1}=\frac{1}{7^{2}} =\frac{1}{49} =0.0204081633[/tex]

For question 4:

[tex]\to [(-3)^{2}]^3= (-3)^{2\cdot 3}= (-3)^{6}=729[/tex]

For question 5:

[tex]\to [5 \cdot (-3)]^{2}= 25 \cdot 9=225[/tex]

For question 6:

[tex]\to [(10 \div 5)]^{3}= [(\frac{10}{5})]^{3}=[2]^{3}=8[/tex]

For question 7:

[tex]\to 10^6 \cdot 10^{-4} \cdot 10^2= 10^6 \cdot \frac{1}{10^{4}} \cdot 10^2= 10^2 \cdot 10^2=10^4=10,000[/tex]

For question 8:

[tex]\to (-4)^{-5}=\frac{1}{(-4)^{5}}=- \frac{1}{1,024}=-0.0009765625[/tex]

For question 9:

[tex]\to \frac{2^3}{2^4}= \frac{8}{16}=\frac{1}{2}=0.5[/tex]

For question 10:

[tex]\to (-6)^3 \cdot (-6)^5 \cdot (-6)^{-5}= (-6)^3 \cdot (-6)^5 \cdot \frac{1}{(-6)^{5}}= (-6)^3 =-216[/tex]

Ariel has 20 bouncy balls. He later then sold them for a price of $2 per ball to his best friend, Mike Hawk. Later, Ariel has gotten back 1/69 of his original amount of bouncy balls and got 20 more. How many bouncy balls does Ariel have?

Answers

ariel has 40 balls left

Answer:

420

Step-by-step explanation:

Which graph represents the solution of x2 + 9y2 ≤ 81 and y2 + 2 < x? On a coordinate plane, an ellipse has center (0, 0) and goes through (3, 0), (0, negative 9), (negative 3, 0), and (0, 9). A parabola opens to the right and goes through (6, 2), has vertex (2, 0), and goes through (6, negative 2). Everything inside of the ellipse and outside of the parabola is shaded. On a coordinate plane, an ellipse has center (0, 0) and goes through (3, 0), (0, negative 9), (negative 3, 0), and (0, 9). A parabola opens to the right and goes through (6, 2), has vertex (2, 0), and goes through (6, negative 2). Everything inside of the ellipse and inside of the parabola is shaded. On a coordinate plane, an ellipse has center (0, 0) and goes through (9, 0), (0, negative 3), (negative 9, 0) and (0, 3). A parabola opens to the right and goes through (6, 2), has vertex (2, 0), and goes through (6, negative 2). Everything inside of the ellipse and inside of the parabola is shaded. On a coordinate plane, an ellipse has center (0, 0) and goes through (9, 0), (0, negative 3), (negative 9, 0) and (0, 3). A parabola opens to the right and goes through (6, 2), has vertex (2, 0), and goes through (6, negative 2). Everything inside of the ellipse and outside of the parabola is shaded.

Answers

9514 1404 393

Answer:

  C

Step-by-step explanation:

The ellipse y-intercepts are ±3, the x-intercepts are ±9, eliminating choices A and B. The parabola is shaded inside, eliminating choice D.

The correct choice is the third one.

álgebra 1 solve -7 + 18(17h + 19)

Answers

Answer:  =360h+335

Step-by-step explanation:

Answer:

306h + 281

Step-by-step explanation:

-7 + 18(17h + 19)

-7 + 306h + 288

306h + 281

Other Questions
Julie and Mona know that that Earths average distance from the Sun is approximately 93 million miles and it takes 1 year to complete an orbit of the Sun. A new asteroid has been discovered orbiting the Sun at an average distance of 1,488 million miles. How long will it take for the asteroid, in Earth years, to complete one orbit of the Sun. [tex]find \: the \: volume \: of \: cylinder \\ \\ radius \: = 4 \: cm \\ \\ height \: = \: 10 \: cm \: \\ \\ [/tex] Help plsssssss ,it would mean a lot thankyou What is the law of multiple proportions? A. The proportion of elements to compounds is constant.B. All elements are found in equal proportions in nature.C. Different compounds may contain the same elements but may have different ratios of those elements. D. All compounds contain the same elements in the same proportions. Which of the following was a mainreason for Adolf Hitler deciding to takeover Austria first?A. Austria was Hitler's home nation.B. Austria had declared war on Germany.C. Austria had been a German enemy in World War I.D. Austria asked Hitler to annex them into Germany. Which of the following financial functions can you use to calculate the payments to repay your loan Qual o smbolo do Manguebeat? Explique?Me ajudem plsVale 11 pontos Write a program that launches 1,000 threads. Each thread adds 1 to a variable sum that initially is 0. You need to pass sum by reference to each thread. In order to pass it by reference, define an Integer wrapper object to hold sum. Run the program with and without synchronization to see its effect. Submit the code of the program and your comments on the execution of the program. A(n) __________ opening involves actually providing a demonstration of a good or service as soon as a salesperson meets the prospect. Study the past to learn about cultures and what they valued How did the Tet Offensive affect the Viet Cong during the Vietnam War?It allowed them to enjoy a resounding military victory.It led to the loss of more than 50,000 of their troops.It decreased their morale because it led to defeat at the hands of the NVA.It led them to develop new tactics to combat guerrilla warfare. the ratio of 10:25 in its simplest form? The impacts of colonialism in Africa?Pls anyone I need it full details Suppose a tank contains 400 gallons of salt water. If pure water flows into the tank at the rate of 7 gallons per minute and the mixture flows out at the rate of 3 gallons per minute, how many pounds of salt will remain in the tank after 16 minutes if 28 pounds of salt are in the mixture initially? (Give your answer correct to at least three decimal places.) At the end of each quarter, Patti deposits $1,100 into an account that pays 12% interest compounded quarterly. How much will Patti have in the account in 4 years 4. An object is thrown from from the ground upward with an initial speed of 3.75 m/s. How long will the object be in the air before it lands on the ground? Assume that the east coast of South America and the west coast of Africa are separated by an average distance of 4,500 km. Assume also that global positioning system (GPS) measurements indicate that these continents are now moving apart at a rate of 3.75 cm/year. If you could assume that this rate has been constant over geological time, how long ago were these two continents joined together as part of a supercontinent Rewrite the sentences using possessive adjectives. 1. El profesor tiene pluma.2. T tienes mochilas.3. Los turistas tienen mapa.4. Usted tiene cuadernos.5. Nosotros tenemos papelera.HELP WILL GIVE BRAINLIEST!!!(i need this legit rn so pls help) Give reasons: a) Cocoons are boiled in hot water. b) Camel wool considered a natural health product. Andrew wants to build a square garden and needs to determine how much area he has for planting the perimeter of the garden is between 12 and 14 feet what is the range if the possible areas